Wie können wir die Wahrscheinlichkeit begrenzen, dass eine Zufallsvariable maximal ist?


21

Angenommen, wir haben unabhängige Zufallsvariablen , , mit endlichen Mitteln und Varianzen , , . Ich suche nach verteilungsfreien Grenzen für die Wahrscheinlichkeit, dass größer ist als alle anderen , .NX1Xnμ1μNσ12σN2XiXNXjji

Mit anderen Worten, wenn wir der Einfachheit halber annehmen, dass die Verteilungen von stetig sind (so dass ), suche ich nach Grenzen für: Wenn N = 2 , können wir Chebyshevs Ungleichung verwenden, um zu erhalten: \ P (X_1 = \ max_j X_j) = \ P (X_1> X_2) \ leq \ frac {\ sigma_1 ^ 2 + \ sigma_2 ^ 2} {\ sigma_1 ^ 2 + \ sigma_2 ^ 2 + (\ mu_1 - \ mu_2) ^ 2} \ enspace. Ich würde gerne ein paar einfache (nicht unbedingt enge) Grenzen für allgemeines N finden , aber ich war nicht in der Lage, (ästhetisch) ansprechende Ergebnisse für allgemeines N zu finden .XiP(Xi=Xj)=0

P(Xi=maxjXj).
N=2N N
P(X1=maxjXj)=P(X1>X2)σ12+σ22σ12+σ22+(μ1μ2)2.
NN

Bitte beachten Sie, dass die Variablen nicht als iid angenommen werden. Anregungen oder Hinweise auf verwandte Arbeiten sind willkommen.


Update: Denken Sie daran, dass \ mu_j \ geq \ mu_i angenommen wirdμjμi . Wir können dann die obige Schranke verwenden, um zu kommen:

P(Xi=maxjXj)minj>iσi2+σj2σi2+σj2+(μjμi)2σi2+σN2σi2+σN2+(μNμi)2.
Dies impliziert:
(μNμi)P(Xi=maxjXj)(μNμi)σi2+σN2σi2+σN2+(μNμi)212σi2+σN2.
Dies impliziert wiederum:
i=1NμiP(Xi=maxjXj)μNN2i=1N1(σi2+σN2).
Ich frage mich jetzt, ob diese Schranke zu etwas verbessert werden kann, das nicht linear von N abhängt N. Zum Beispiel gilt Folgendes:
i=1NμiP(Xi=maxjXj)μNi=1Nσi2?
Und wenn nicht, was könnte ein Gegenbeispiel sein?

3
Diese Grenze kann enger sein, wenn Sie den Index , der Ihnen die kleinere obere Grenze anstelle von . Beachten Sie, dass dieser Wert sowohl vom Mittelwert als auch von der Varianz abhängt. NjN

5
@MichaelChernick: Ich glaube nicht, dass das richtig ist. Nehmen wir zum Beispiel an, wir haben drei Gleichverteilungen für . Dann, wenn ich mich nicht irre, istP ( X 1 < max j X j ) = 2 / 3[0,1]P(X1<maxjXj)=2/3 , wohingegen . Ich weiß nicht, ob Sie schreiben , aber dasselbe Beispiel zeigt, dass es immer noch nicht gültig ist. P ( X i > max j X j )P(X1<X2)=P(X1<X3)=1/2P(Xi>maxjXj)
MLS

2
@Michael: Das stimmt leider immer noch nicht. Die Ereignisse für festes sind nicht unabhängig. Aj={Xi>Xj} i
Kardinal

2
@ Cardinal: Unter anderem handelt es sich um mehrarmige Banditen. Wenn Sie einen Arm basierend auf vorherigen Belohnungen auswählen, wie groß ist die Wahrscheinlichkeit, dass Sie den besten Arm ausgewählt haben (das wäre in der obigen Notation), und können wir den erwarteten Verlust für die Auswahl eines Sub- begrenzen -optimaler Arm? P(XN=maxjXj)
MLS

2
Crossposted zu MathOverflow: mathoverflow.net/questions/99313
Kardinal

Antworten:


1

Sie können die multivariate Chebyshev-Ungleichung verwenden.

Fall mit zwei Variablen

Für eine einzelne Situation, vs , komme ich zur gleichen Situation wie Jochens Kommentar am 4. November 2016X 2X1X2

1) Wenn dann P ( X 1 > X 2 ) ( σ 2 1 + σ 2 2 ) / ( μ 1 - μ 2 ) 2μ1<μ2P(X1>X2)(σ12+σ22)/(μ1μ2)2

(Und ich frage mich auch über Ihre Ableitung)

Herleitung von Gleichung 1

  • mit der neuen VariablenX1X2
  • Transformiere es so, dass es den Mittelwert bei Null hat
  • den absoluten Wert nehmen
  • Anwendung der Ungleichung von Chebyshev

P(X1>X2)=P(X1X2>0)=P(X1X2(μ1μ2)>(μ1μ2))P(|X1X2(μ1μ2)|>μ2μ1)σ(X1X2(μ1μ2))2(μ2μ1)2=σX12+σX22(μ2μ1)2

Multivariater Fall

Die Ungleichung in Gleichung (1) kann durch Anwenden auf mehrere transformierte Variablen für jedes in einen multivariaten Fall geändert werden (beachte, dass diese korreliert sind).i < n(XnXi)i<n

Eine Lösung für dieses Problem (multivariat und korreliert) wurde von I. Olkin und JW Pratt beschrieben. 'A Multivariate Tchebycheff Inequality' in den Annals of Mathematical Statistics, Band 29, Seiten 226-234 http://projecteuclid.org/euclid.aoms/1177706720

Beachten Sie den Satz 2.3

P(|yi|kiσi for some i)=P(|xi|1 for some i)(u+(ptu)(p1))2p2

in welcher die Anzahl der Variablen ist, und .pt=ki2u=ρij/(kikj)

Satz 3.6 liefert eine engere Grenze, ist jedoch weniger einfach zu berechnen.

Bearbeiten

Mit der multivariaten Cantelli-Ungleichung kann eine schärfere Grenze gefunden werden . Dass die Ungleichheit ist die Art , die man verwendete , vor und vorausgesetzt Sie mit der Grenze die ist , schärfer als .(σ12+σ22)/(σ12+σ22+(μ1μ2)2)(σ12+σ22)/(μ1μ2)2

Ich habe mir nicht die Zeit genommen, den gesamten Artikel zu studieren, aber trotzdem können Sie hier eine Lösung finden:

AW Marshall und I. Olkin "Eine einseitige Ungleichung des Chebyshev-Typs" in Annals of Mathematical Statistics, Band 31, Seiten 488-491 https://projecteuclid.org/euclid.aoms/1177705913

(späterer Hinweis: Diese Ungleichung ist für gleiche Korrelationen und nicht ausreichend hilfreich. Aber Ihr Problem, die schärfste Grenze zu finden, ist auf jeden Fall gleich der allgemeineren multivariaten Cantelli-Ungleichung. Ich wäre überrascht, wenn die Lösung nicht existiert.)


Können Sie eine klare Aussage zur multivariaten Chebyshev-Ungleichung machen?
Whuber

1
Ich habe die Lösung mit dem gesamten Theorem überarbeitet.
Sextus Empiricus

-1

Ich habe einen Satz gefunden, der Ihnen helfen könnte und der versucht, ihn an Ihre Bedürfnisse anzupassen. Angenommen, Sie haben:

exp(tE(max1inXi))

Dann erhalten wir durch Jensens Ungleichung (da exp (.) Eine konvexe Funktion ist):

exp(tE(max1inXi))E(exp(tmax1inXi))=E(max1in exp(tXi))i=1nE(exp(tXi)

Nun müssen Sie für die momentane Erzeugungsfunktion Ihrer Zufallsvariablen einstecken (da dies nur die Definition der mgf ist) Vereinfachen Sie Ihren Begriff), nehmen Sie diesen Begriff und nehmen das Protokoll und dividieren Sie es durch t, ​​so dass Sie eine Aussage über den Begriff . Dann können Sie t mit einem beliebigen Wert auswählen (am besten, damit der Term klein ist und die Schranke eng ist).exp(tXiXiE(max1inXi)

Dann haben Sie eine Aussage über den erwarteten Wert des Maximums über n rvs. Um nun eine Aussage über die Wahrscheinlichkeit zu erhalten, dass das Maximum dieser rvs von diesem erwarteten Wert abweicht, können Sie einfach Markovs Ungleichung verwenden (vorausgesetzt, Ihr rv ist nicht negativ) oder ein anderes, spezifischeres rv, das auf Ihr bestimmtes rv zutrifft.

Durch die Nutzung unserer Website bestätigen Sie, dass Sie unsere Cookie-Richtlinie und Datenschutzrichtlinie gelesen und verstanden haben.
Licensed under cc by-sa 3.0 with attribution required.